subject
Mathematics, 31.03.2021 18:40 brookesquibbs

Point P on a coordinate plane, located 3 units to the left of the vertical axis and 2 units below the horizontal axis. A. (–3, –2)
B. (–3, 2)
C. (–2, –3)
D. (–3, –3)

ansver
Answers: 2

Another question on Mathematics

question
Mathematics, 21.06.2019 14:30
Me #1 using vocabulary words, including the word β€œcongruent”, state why the measure of angle d is 120 degrees. #2 what is the measure of angle e and why?
Answers: 1
question
Mathematics, 21.06.2019 20:00
How can you construct perpendicular lines and prove theorems about perpendicular lines
Answers: 3
question
Mathematics, 21.06.2019 20:30
William invested $5000 in an account that earns 3.8% interest, compounded annually. the formula for compound interest is a(t) = p(1 + i)t. how much did william have in the account after 6 years? (apex)
Answers: 2
question
Mathematics, 22.06.2019 02:50
1. how do you find p(a and b) if a and b are dependent events for two or more actions, such as selecting two candies and drawing p(green, and then pink)?
Answers: 1
You know the right answer?
Point P on a coordinate plane, located 3 units to the left of the vertical axis and 2 units below th...
Questions
question
Mathematics, 23.08.2021 07:50
question
Mathematics, 23.08.2021 07:50
question
Social Studies, 23.08.2021 07:50
question
Mathematics, 23.08.2021 08:00
Questions on the website: 13722363